2014 dxdy logo

Научный форум dxdy

Математика, Физика, Computer Science, Machine Learning, LaTeX, Механика и Техника, Химия,
Биология и Медицина, Экономика и Финансовая Математика, Гуманитарные науки




Начать новую тему Ответить на тему
 
 Иррациональные неравенства от четырёх переменных
Сообщение17.01.2021, 15:55 
Заслуженный участник


26/06/07
1929
Tel-aviv
Пусть $a,$ $b$, $c$ и $d$ неотрицательные числа, для которых $a+b+c+d=4$. Докажите, что:
1. $$\sqrt{\frac{a}{b+1}}+\sqrt{\frac{b}{c+1}}+\sqrt{\frac{c}{d+1}}+\sqrt{\frac{d}{a+1}}\leq3;$$
2. $$\sqrt{\frac{a}{b+3}}+\sqrt{\frac{b}{c+3}}+\sqrt{\frac{c}{d+3}}+\sqrt{\frac{d}{a+3}}\leq2;$$
3. $$\sqrt[4]{\frac{a}{10b+1}}+\sqrt[4]{\frac{b}{10c+1}}+\sqrt[4]{\frac{c}{10d+1}}+\sqrt[4]{\frac{d}{10a+1}}\leq6\sqrt[4]{\frac{2}{55}};$$
4. $$\sqrt[5]{\frac{a}{5b+1}}+\sqrt[5]{\frac{b}{5c+1}}+\sqrt[5]{\frac{c}{5d+1}}+\sqrt[5]{\frac{d}{5a+1}}\leq\sqrt[5]{200};$$
5. Пусть $a,$ $b$, $c$ и $d$ неотрицательные числа, для которых $a+b+c+d=5$. Докажите, что:
$$\sqrt[3]{\frac{a}{b+1}}+\sqrt[3]{\frac{b}{c+1}}+\sqrt[3]{\frac{c}{d+1}}+\sqrt[3]{\frac{d}{a+1}}\leq\sqrt[3]{36}.$$

(PS)

Все оценки точные. Первая и пятая задачи имеют красивое решение в одну строчку. Вторая также имеет простое решение, но не в одну строчку.

 Профиль  
                  
 
 Re: Иррациональные неравенства от четырёх переменных
Сообщение18.01.2021, 08:29 
Заблокирован


16/04/18

1129
Если бы у бабушки... В смысле если бы в последней задаче сумма была бы не 5, а 4, то она следовала бы из первой и неравенства о средних.

 Профиль  
                  
 
 Re: Иррациональные неравенства от четырёх переменных
Сообщение18.01.2021, 11:23 
Заблокирован


16/04/18

1129
Для 1) пока получилось простыми средствами с худшей постоянной справа=4.

 Профиль  
                  
 
 Re: Иррациональные неравенства от четырёх переменных
Сообщение04.02.2021, 06:19 
Модератор
Аватара пользователя


11/01/06
5660
arqady, напомнило вот эти ваши неравенства:
arqady в сообщении #106301 писал(а):
Пусть $a,$ $b$ и $c$ неотрицательные числа, никакие два из которых не равны нулю. Докажите, что
1) $$\sqrt{1+\frac{48a}{b+c}}+\sqrt{1+\frac{48b}{a+c}}+\sqrt{1+\frac{48c}{a+b}}\geq15.$$
2)$$\sqrt{\frac{a}{4a+5b}}+\sqrt{\frac{b}{4b+5c}}+\sqrt{\frac{c}{4c+5a}}\leq1.$$
3)$$\sqrt[3]{\frac{a}{2b+25c}}+\sqrt[3]{\frac{b}{2c+25a}}+\sqrt[3]{\frac{c}{2a+25b}}\geq1.$$
Имеется мощный совершенно элементарный общий метод доказательства. Сам нашёл! :D

Неравенства из этой темы тем же общим методом доказываются?

 Профиль  
                  
 
 Re: Иррациональные неравенства от четырёх переменных
Сообщение04.02.2021, 11:28 
Заслуженный участник


26/06/07
1929
Tel-aviv
Нет. Эти (первое и второе принадлежат Василе Кыртоаже) неравенства доказываются от противного(могу показать, как конкрентно). Уже для третьего неравенства это очень трудно реализовать.
Но можно воспользоваться ещё теоремой о право выпуклой функции Василе Кыртоаже. Мои неравенства доказываются другим способом.
Кстати, недавно (в этот понедельник) выяснилось, что и следующее неравенство можно доказать в одну строчку.
Пусть $a,$ $b$, $c$ и $d$ неотрицательные числа, для которых $a+b+c+d=4$. Докажите, что:
$$\sqrt{\frac{a}{b+2}}+\sqrt{\frac{b}{c+2}}+\sqrt{\frac{c}{d+2}}+\sqrt{\frac{d}{a+2}}\leq\frac{4}{\sqrt3}$$

 Профиль  
                  
 
 Re: Иррациональные неравенства от четырёх переменных
Сообщение12.02.2021, 19:26 
Заслуженный участник


03/01/09
1683
москва
В 1) получилось доказать лишь более слабое неравенство.
1) Применим к первому и третьему, а также ко второму и четвертому слагаемым суммы $S$ неравенство К.-Б., получим:$$S\leq \sqrt {(a+c)(\frac 1{b+1}+\frac 1{d+1})}+\sqrt {(b+d)(\frac 1{a+1}+\frac 1{c+1})}$$Обозначим $a+c=x, b+d=4-x, \dfrac 1{a+1}+\dfrac 1{c+1}=\dfrac {x+2}{ac+x+1}\leq \dfrac {x+2}{x+1}$. Аналогично получим: $\dfrac 1{b+1}+\dfrac 1{d+1}\leq \dfrac {6-x}{5-x}$
Таким образом:$$S\leq \sqrt {\frac {x(6-x)}{5-x}}+\sqrt {\frac {(4-x)(x+2)}{x+1}}\eqno (1)$$
Правая часть неравенства (1) достигает наибольшего значения при $x=2$, поэтому $S\leq 4\sqrt {\frac 23}\approx 3.3$
Таким же способом можно получить оценку для суммы 2) и вообще для сумм вида $S_n=\sqrt {\dfrac a{b+n}}+\cdots$, где $a+b+c+d=4$.
В связи с этим можно предположить, что в общем случае выполняется неравенство: $S_n\leq \dfrac 4{\sqrt {n+1}}$,(это значение достигается при $a=b=c=d=1$,и к нему асимптотически приближается верхняя оценка для $S_n$) т.е., например, неравенство 1) можно было бы еще улучшить до $S\leq 2\sqrt 2$.

 Профиль  
                  
 
 Re: Иррациональные неравенства от четырёх переменных
Сообщение12.02.2021, 23:09 
Аватара пользователя


26/02/14
497
so dna
mihiv в сообщении #1504873 писал(а):
В связи с этим можно предположить, что в общем случае выполняется неравенство: $S_n\leq \dfrac 4{\sqrt {n+1}}$,(это значение достигается при $a=b=c=d=1$,и к нему асимптотически приближается верхняя оценка для $S_n$) т.е., например, неравенство 1) можно было бы еще улучшить до $S\leq 2\sqrt 2$.
Это неверно, т.к. равенство в 1) достигается при $a=c=1+\frac{2\sqrt{5}}{5}, b=d=1-\frac{2\sqrt{5}}{5}$ и его нельзя улучшить путём уменьшения правой части на константу.

 Профиль  
                  
 
 Re: Иррациональные неравенства от четырёх переменных
Сообщение12.02.2021, 23:43 
Заслуженный участник


03/01/09
1683
москва
Rak so dna
Меня ввело в заблуждение то, что для $n=2,3$ неравенства выполняются.

 Профиль  
                  
 
 Re: Иррациональные неравенства от четырёх переменных
Сообщение13.02.2021, 17:30 
Заблокирован


16/04/18

1129
Вопрос: это задача на применение не общеизвестной специальной теоремы про выпуклые/вогнутые функции? Или есть решение без этого, в мейнстриме?

 Профиль  
                  
 
 Re: Иррациональные неравенства от четырёх переменных
Сообщение13.02.2021, 18:48 
Заслуженный участник


26/06/07
1929
Tel-aviv
В моём однострочечном решении я пользовался AM-GM и Jensen.

 Профиль  
                  
 
 Re: Иррациональные неравенства от четырёх переменных
Сообщение16.04.2021, 21:31 
Аватара пользователя


26/02/14
497
so dna
arqady в сообщении #106301 писал(а):
Пусть $a,$ $b$ и $c$ неотрицательные числа, никакие два из которых не равны нулю. Докажите, что
1) $$\sqrt{1+\frac{48a}{b+c}}+\sqrt{1+\frac{48b}{a+c}}+\sqrt{1+\frac{48c}{a+b}}\geq15.$$
Обозначим $\frac{2a}{b+c}=A, \frac{2b}{c+a}=B, \frac{2c}{a+b}=C,$ тогда достаточно доказать $\sum\limits_{cyc}\sqrt{24A+1}\geqslant15,$ для неотрицательных $A,B,C$ таких, что $\sum\limits_{cyc}\frac{1}{A+2}=1.$
От противного: предположим, что существуют такие $A_0,B_0,C_0\geqslant0, \quad \sum\limits_{cyc}\frac{1}{A_0+2}=1,$ что $\sum\limits_{cyc}\sqrt{24A_0+1}<15.$ Тогда, существует такое $\theta>1$, при котором $\sum\limits_{cyc}\sqrt{24\theta A_0+1}=15.$ Обозначим $A_1=\theta A_0, B_1=\theta B_0, C_1=\theta C_0,$ тогда имеем: $\sum\limits_{cyc}\sqrt{24A_1+1}=15; \quad \sum\limits_{cyc}\frac{1}{A_1+2} = \sum\limits_{cyc}\frac{1}{\theta A_0+2} < 1; \quad A_1,B_1,C_1\geqslant0.$ Теперь, поскольку $\sum\limits_{cyc}\frac{1}{A_1+2} < 1 \Leftrightarrow A_1B_1C_1+A_1B_1+B_1C_1+C_1A_1 > 4,$ для получения противоречия достаточно доказать, что для любых $X,Y,Z\geqslant0,$ для которых $\sum\limits_{cyc}\sqrt{24X+1}=15,$ выполняется неравенство $XYZ+XY+YZ+ZX \leqslant 4.$ Докажем это:
Для любых $X,Y,Z\geqslant0,$ найдутся такие $x,y,z\geqslant0,$ что $X=\frac{x^2+x}{6}, Y=\frac{y^2+y}{6}, Z=\frac{z^2+z}{6}$, поэтому достаточно доказать для неотрицательных $x,y,z,$ таких, что $x+y+z=6$ неравенство $$f(x,y,z)=4-\frac{1}{6^3}xyz(x+1)(y+1)(z+1)-\frac{1}{6^2}\sum\limits_{cyc}xy(x+1)(y+1)\geqslant0$$Имеем:
$f(x,y,z)\stackrel{AM-GM}{\geqslant}4-\frac{1}{6^3}xyz\left(\frac{x+y+z+3}{3}\right)^3-\frac{1}{6^2}\sum\limits_{cyc}xy(x+1)(y+1)=$
$=4\left(\frac{x+y+z}{6}\right)^4-\frac{1}{8}xyz\left(\frac{x+y+z}{6}\right)-\frac{1}{6^2}\sum\limits_{cyc}xy\left(x+\left(\frac{x+y+z}{6}\right)\right)\left(y+\left(\frac{x+y+z}{6}\right)\right)=$
$=\frac{1}{6^4}\sum\limits_{cyc}(x-y)^2(13xy+2(x+y-z)^2) \geqslant0.$

arqady в сообщении #106301 писал(а):
Пусть $a,$ $b$ и $c$ неотрицательные числа, никакие два из которых не равны нулю. Докажите, что
2)$$\sqrt{\frac{a}{4a+5b}}+\sqrt{\frac{b}{4b+5c}}+\sqrt{\frac{c}{4c+5a}}\leq1.$$

$\Leftrightarrow \sum\limits_{cyc}\sqrt{\frac{1}{4+5\frac{b}{a}}} \leqslant1  \Leftrightarrow A=\frac{b}{a}, B = \frac{c}{b}, C=\frac{a}{c}, \quad \sum\limits_{cyc}\sqrt{\frac{1}{4+5A}}\leqslant1$
$ABC = 1$ и достаточно доказать это для $A,B,C\geqslant0$
От противного: предположим, что существуют такие $A_0,B_0,C_0\geqslant0, \quad A_0B_0C_0=1,$ что $\sum\limits_{cyc}\sqrt{\frac{1}{4+5A_0}}>1.$ Тогда, существует такое $\theta > 1$, при котором $\sum\limits_{cyc}\sqrt{\frac{1}{4+5\theta A_0}}=1.$ Обозначим $A_1=\theta A_0, B_1=\theta B_0, C_1=\theta C_0,$ тогда имеем: $\sum\limits_{cyc}\sqrt{\frac{1}{4+5A_1}}=1; \quad  A_1B_1C_1=\theta^3A_0B_0C_0>1; \quad A_1,B_1,C_1\geqslant0.$ Для получения противоречия достаточно доказать, что для любых $X,Y,Z\geqslant0,$ для которых $\sum\limits_{cyc}\sqrt{\frac{1}{4+5X}}=1,$ выполняется неравенство $XYZ \leqslant 1.$
Докажем это:
Для любых $X,Y,Z\geqslant0,$ найдутся такие $p,q,r\geqslant0,$ что $X=\frac{p^2+4p}{5}, Y=\frac{q^2+4q}{5}, Z=\frac{r^2+4r}{5}$, поэтому достаточно доказать для неотрицательных $p,q,r,$ таких, что $\sum\limits_{cyc}\frac{1}{p+2}=1$ неравенство $\prod\limits_{cyc}p(p+4)\leqslant5^3.$
$\Leftrightarrow \prod\limits_{cyc}\frac{2x}{y+z}\left(\frac{2x}{y+z}+4\right)\leqslant5^3; \quad x,y,z\geqslant0 $

(почему неотрицательные?)

покажем, что $x,y,z$ можно считать неотрицательными. Имеем:
$$\begin{cases}
p=\frac{2x}{y+z}\geqslant0\\
q=\frac{2y}{z+x}\geqslant0\\
r=\frac{2z}{x+y}\geqslant0
\end{cases} \Rightarrow 
\begin{cases}
x(y+z)\geqslant0\\
y(z+x)\geqslant0\\
z(x+y)\geqslant0
\end{cases} \Rightarrow 
\begin{cases}
xy=\frac{x^2y^2z^2+x^2y^2\big(y(z+x)+x(y+z)\big)}{x^2y^2+y(z+x)\cdot x(y+z)}\geqslant0\\
yz=\frac{x^2y^2z^2+y^2z^2\big(z(x+y)+y(z+x)\big)}{y^2z^2+z(x+y)\cdot y(z+x)}\geqslant0\\
zx=\frac{x^2y^2z^2+z^2x^2\big(x(y+z)+z(x+y)\big)}{z^2x^2+x(y+z)\cdot z(x+y)}\geqslant0\\
\end{cases} 
$$ поэтому $x,y,z$ одного знака, но $-x,-y,-z$ соответствуют $x,y,z$ а значит мы имеем право считать их неотрицательными

$\Leftrightarrow \sum\limits_{cyc}z(x-y)^2(125z^3+125z^2(x+y)+101xyz+122xy(x+y))\geqslant0$

arqady в сообщении #106301 писал(а):
Пусть $a,$ $b$ и $c$ неотрицательные числа, никакие два из которых не равны нулю. Докажите, что
3)$$\sqrt[3]{\frac{a}{2b+25c}}+\sqrt[3]{\frac{b}{2c+25a}}+\sqrt[3]{\frac{c}{2a+25b}}\geq1.$$

$\Leftrightarrow \sum\limits_{cyc}\sqrt[3]{\frac{1}{2\frac{b}{c}\frac{c}{a}+25\frac{c}{a}}} \geqslant1 \Leftrightarrow A=\frac{a}{b}, B = \frac{b}{c}, C=\frac{c}{a}, \quad \sum\limits_{cyc}\sqrt[3]{\frac{1}{2BC+25C}}\geqslant1$
$ABC = 1; \quad A,B,C>0$
От противного: предположим, что существуют такие $A_0,B_0,C_0>0, \quad A_0B_0C_0=1,$ что $\sum\limits_{cyc}\sqrt[3]{\frac{1}{2B_0C_0+25C_0}}<1.$ Тогда, существует такое $0<\theta < 1$, при котором $\sum\limits_{cyc}\sqrt[3]{\frac{1}{2\theta^2B_0C_0+25\theta C_0}}=1.$
Обозначим $A_1=\theta A_0, \quad B_1=\theta B_0, \quad C_1=\theta C_0,$ тогда имеем: $\sum\limits_{cyc}\sqrt[3]{\frac{1}{2B_1C_1+25C_1}}=1; \quad  A_1B_1C_1=\theta^3A_0B_0C_0<1; \quad A_1,B_1,C_1>0.$ Для получения противоречия достаточно доказать, что для любых $X,Y,Z>0,$ для которых $\sum\limits_{cyc}\sqrt[3]{\frac{1}{2YZ+25Z}}=1,$ выполняется неравенство $XYZ \geqslant 1.$
Докажем это:
Пусть $X(2Z+25)=\frac{1}{x^3}, \quad Y(2X+25)=\frac{1}{y^3}, \quad Z(2Y+25)=\frac{1}{z^3}; \quad x,y,z>0$ Тогда

$2(XY+YZ+ZX)+25(X+Y+Z)=\frac{1}{x^3}+\frac{1}{y^3}+\frac{1}{z^3}$ и

$XYZ(2X+25)(2Y+25)(2Z+25)=$

$=8(XYZ)^2+XYZ\big(50(2(XY+YZ+ZX)+25(X+Y+Z))+25^3\big)=$

$=8(XYZ)^2+XYZ\left(50\left(\frac{1}{x^3}+\frac{1}{y^3}+\frac{1}{z^3}\right)+25^3\right)=\frac{1}{x^3y^3z^3},$ отсюда

$XYZ=\frac{1}{16}\sqrt{625\left(2\left(\frac{1}{x^3}+\frac{1}{y^3}+\frac{1}{z^3}\right)+625\right)^2+\frac{32}{x^3y^3z^3}}-\frac{25}{16}\left(2\left(\frac{1}{x^3}+\frac{1}{y^3}+\frac{1}{z^3}\right)+625\right)$
Поэтому для $x+y+z=1$ надо доказать, что
$$\frac{1}{16}\sqrt{625\left(2\left(\frac{1}{x^3}+\frac{1}{y^3}+\frac{1}{z^3}\right)+625\right)^2+\frac{32}{x^3y^3z^3}}\geqslant\frac{25}{16}\left(2\left(\frac{1}{x^3}+\frac{1}{y^3}+\frac{1}{z^3}\right)+625\right)+1$$

$\Leftrightarrow 1-50(x^3y^3+y^3z^3+z^3x^3)-15633x^3y^3z^3\geqslant0, \quad x+y+z=1$

$\Leftrightarrow (x+y+z)^9-50(x+y+z)^3(x^3y^3+y^3z^3+z^3x^3)-15633x^3y^3z^3\geqslant0$

(SOS)

$LHS = \frac{1}{2} \sum\limits_{cyc}(y-z)^2\big(966x^4yz(y+z)+4701x^3y^2z^2+\\(111(y^5+z^5)+870yz(y^3+z^3)+2385y^2z^2(y+z))x^2+(72yz(y^4+z^4)+636y^3z^3)x+y^7+z^7+20yz(y^5+z^5)+48y^3z^3(y+z)\big)\geqslant0$

Можно ещё BW

 Профиль  
                  
 
 Re: Иррациональные неравенства от четырёх переменных
Сообщение30.05.2021, 10:38 
Аватара пользователя


26/02/14
497
so dna
arqady в сообщении #1501573 писал(а):
Пусть $a,$ $b$, $c$ и $d$ неотрицательные числа, для которых $a+b+c+d=4$. Докажите, что:...
Скорее всего верно следующее:

Определим многочлены $P_{n}(t)$ и $Q_{n}(t)$ из разложения
$2^{n-1}t^{\left[\frac{n+1}{2}\right]}P_{n}(t)+2^{n-1}t^{\left[\frac{n}{2}\right]}Q_{n}(t)\sqrt{t(t+4)} = \left(t+\sqrt{t(t+4)}\right)^n,$
тогда для неотрицательных $k,$ $a,$ $b,$ $c$ и $d,$ для которых $a+b+c+d=S$ выполняется:
____

$1.\quad m=2n:$
$$\sqrt[m]{\frac{a}{b+k}}+\sqrt[m]{\frac{b}{c+k}}+\sqrt[m]{\frac{c}{d+k}}+\sqrt[m]{\frac{d}{a+k}}\leq\begin{cases}
2(\gamma+2)\sqrt[m]{\frac{P_{2n-1}(\gamma)}{P_{2n+1}(\gamma)}},\quad S\geqslant 2k(2n-1)\\
4\sqrt[m]{\frac{S}{S+4k}},\quad S\leqslant k(2n-1)
\end{cases}$$ где $\gamma$ - неотрицательный корень уравнения $P_{2n-1}(\gamma)Q_{2n+1}(\gamma)=\frac{S}{2k},$
____

$2.\quad m=4n-1:$
$$\sqrt[m]{\frac{a}{b+k}}+\sqrt[m]{\frac{b}{c+k}}+\sqrt[m]{\frac{c}{d+k}}+\sqrt[m]{\frac{d}{a+k}}\leq\begin{cases}
2\sqrt[m]{\frac{P_{2n-1}(\gamma)}{Q_{2n}(\gamma)}(\gamma+4)^{2n-1}},\quad S\geqslant 4k(2n-1)\\
4\sqrt[m]{\frac{S}{S+4k}},\quad S\leqslant 2k(2n-1)
\end{cases}$$ где $\gamma$ - неотрицательный корень уравнения $P_{2n-1}(\gamma)P_{2n}(\gamma)=\frac{S}{2k},$
____

$3.\quad m=4n+1:$
$$\sqrt[m]{\frac{a}{b+k}}+\sqrt[m]{\frac{b}{c+k}}+\sqrt[m]{\frac{c}{d+k}}+\sqrt[m]{\frac{d}{a+k}}\leq\begin{cases}
2\sqrt[m]{\frac{Q_{2n}(\gamma)}{P_{2n+1}(\gamma)}(\gamma+4)^{2n+1}},\quad S\geqslant 8kn\\
4\sqrt[m]{\frac{S}{S+4k}},\quad S\leqslant 4kn
\end{cases}$$ где $\gamma$ - неотрицательный корень уравнения $(\gamma+4)Q_{2n}(\gamma)Q_{2n+1}(\gamma)=\frac{S}{2k},$
_______________________________________

$1.\quad m=2n$ тезисное рассуждение:
Рассмотрим функцию $F(x, s) =\sqrt[m]{\frac{x}{s-x+1}}+\sqrt[m]{\frac{s-x}{x+1}}$ на $x,s\geqslant0$

$F'_x\geqslant0 \Leftrightarrow G(x)=(x-s)^{2n-1}(x+1)^{2n+1}-x^{2n-1}(x-s-1)^{2n+1}\leqslant0$

$G(x)=(x-\frac{s}{2})(x-x_1)(x-x_2)R(x)$ где

$x_{1,2}=\frac{s}{2}\pm\frac{P_{2n-1}(\alpha)P_{2n+1}(\alpha)}{2}\sqrt{\frac{\alpha}{\alpha+4}}\geqslant0$

$\alpha$ - неотрицательный корень уравнения $P_{2n-1}(\alpha)Q_{2n+1}(\alpha)=s\Rightarrow s\geqslant 2n-1$

$R(x)$

(не имеет положительных корней)

для $x\geqslant s+1$ проверяем положительность коэффициентов многочлена $G(s+1+t)$
для $s\leqslant x\leqslant s+1$ проверяем положительность коэффициентов многочлена $G(s+\frac{1}{1+t})$
для $0\leqslant x\leqslant s$ проверяем положительность коэффициентов многочленов $R(\frac{s}{2}\pm\frac{\sqrt{s^2-4t}}{2})$

Поэтому уравнение $G(x)=0$ имеет три неотрицательных корня, при $s\geqslant 2n-1$ и один неотрицательный корень при $s\leqslant 2n-1$

Поэтому $$F_{\max}(s)=\begin{cases}
F(x_1,s)=F(x_2,s)=(\alpha+2)\sqrt[m]{\frac{P_{2n-1}(\alpha)}{P_{2n+1}(\alpha)}},\quad s\geqslant2n-1\\
F(\frac{s}{2},s)=2\sqrt[m]{\frac{s}{s+2}},\quad s\leqslant 2n-1
\end{cases}$$
Т.о. $\sqrt[m]{\frac{x}{w+k}}+\sqrt[m]{\frac{w}{x+k}}=F(\frac{x}{k}, \frac{x+w}{k})\leqslant F_{\max}(\frac{x+w}{k})=\begin{cases}
(\alpha+2)\sqrt[m]{\frac{P_{2n-1}(\alpha)}{P_{2n+1}(\alpha)}},\quad x+w\geqslant k(2n-1)\\
2\sqrt[m]{\frac{x+w}{x+w+2k}},\quad x+w\leqslant k(2n-1)
\end{cases}$

где $\alpha$ - неотрицательный корень уравнения $P_{2n-1}(\alpha)Q_{2n+1}(\alpha)=\frac{x+w}{k},$
____

Отметим для всех $t\geqslant 0$:

Утверждение $T_1:$ функция $(\theta_t+2)\sqrt[m]{\frac{P_{2n-1}(\theta_t)}{P_{2n+1}(\theta_t)}}$ является вогнутой

Утверждение $T_2:\quad (\theta_t+2)\sqrt[m]{\frac{P_{2n-1}(\theta_t)}{P_{2n+1}(\theta_t)}}\geqslant F(\frac{t}{2},t)$

тут $\theta_t$ - наибольший (не обязательно неотрицательный) корень уравнения $P_{2n-1}(\theta_t)Q_{2n+1}(\theta_t)=t$
____

Пусть $x,y,z,w$ - это числа $a,b,c,d$ расположенные по возрастанию, тогда
$\sqrt[m]{x}\leqslant\sqrt[m]{y}\leqslant\sqrt[m]{z}\leqslant\sqrt[m]{w}$
$\sqrt[m]{\frac{1}{w+k}}\leqslant\sqrt[m]{\frac{1}{z+k}}\leqslant\sqrt[2n]{\frac{1}{y+k}}\leqslant\sqrt[m]{\frac{1}{x+k}}$ и по перестановочному неравенству

$\sum\limits_{cyc}\sqrt[m]{\frac{a}{b+k}}\leqslant\left(\sqrt[m]{\frac{x}{w+k}}+\sqrt[m]{\frac{w}{x+k}}\right)+\left(\sqrt[2n]{\frac{y}{z+k}}+\sqrt[m]{\frac{z}{y+k}}\right)\leqslant$
$\leqslant F_{\max}(\frac{x+w}{k})+F_{\max}(\frac{y+z}{k})$

Случай $S\leqslant k(2n-1):$
$\begin{cases}
x+w\leqslant S\leqslant k(2n-1)\\
y+z\leqslant S\leqslant k(2n-1)
\end{cases}\Rightarrow F_{\max}(\frac{x+w}{k})+F_{\max}(\frac{y+z}{k})=$
$= 2\sqrt[m]{\frac{x+w}{x+w+2k}}+2\sqrt[m]{\frac{y+z}{y+z+2k}}\stackrel{Jensen}\leqslant 4\sqrt[m]{\frac{\frac{x+y+z+w}{2}}{\frac{x+y+z+w}{2}+2k}}=4\sqrt[m]{\frac{S}{S+4k}}$

Случай $S\geqslant 2k(2n-1):$

$F_{\max}(\frac{x+w}{k})+F_{\max}(\frac{y+z}{k})\stackrel{T_2}\leqslant$

$(\theta_{(x+w)}+2)\sqrt[m]{\frac{P_{2n-1}(\theta_{(x+w)})}{P_{2n+1}(\theta_{(x+w)})}}+(\theta_{(y+z)}+2)\sqrt[m]{\frac{P_{2n-1}(\theta_{(y+z)})}{P_{2n+1}(\theta_{(y+z)})}}\stackrel{T_1\; and\; Jensen}\leqslant $

$(\gamma+2)\sqrt[m]{\frac{P_{2n-1}(\gamma)}{P_{2n+1}(\gamma)}}$

где $\theta_t$ - наибольший (не обязательно неотрицательный) корень уравнения $P_{2n-1}(\theta_t)Q_{2n+1}(\theta_t)=\frac{t}{k},$
$\gamma$ - неотрицательный корень уравнения
$P_{2n-1}(\gamma)Q_{2n+1}(\gamma)=\frac{S}{2k}$ (условие $S\geqslant 2k(2n-1)$ гарантирует такой корень. При остальных $S\geqslant0$ эта оценка неравенства $1.$ сохраняется, но равенство уже не достигается).
_____________________

Оставшиеся случаи $m=4n\pm1$ рассматриваются, в общем аналогично, только уже для функции $F(\frac{s}{1+x^2},s),$ поскольку у $F(x,s)$ проблемы с неотрицательностью одного из корней.

 Профиль  
                  
 
 Re: Иррациональные неравенства от четырёх переменных
Сообщение27.12.2022, 16:31 
Аватара пользователя


26/02/14
497
so dna
arqady в сообщении #1501573 писал(а):
5. Пусть $a,$ $b$, $c$ и $d$ неотрицательные числа, для которых $a+b+c+d=5$. Докажите, что:
$$\sqrt[3]{\frac{a}{b+1}}+\sqrt[3]{\frac{b}{c+1}}+\sqrt[3]{\frac{c}{d+1}}+\sqrt[3]{\frac{d}{a+1}}\leq\sqrt[3]{36}.$$

Воспользуемся:

arqady в сообщении #456475 писал(а):
Пусть $a_i$, $b_i$ и $c_i$, $\alpha$, $\beta$ и $\gamma$ положительны. Тогда согласно Гёльдеру:
$(a_1+a_2+...+a_k)^{\alpha}(b_1+b_2+...+b_k)^{\beta}(c_1+c_2+...+c_k)^{\gamma}\geq\left(\sum\limits_{i=1}^k\left(a_i^{\alpha}b_i^{\beta}\right)^{\frac{1}{\alpha+\beta}}\right)^{\alpha+\beta}\left(\sum\limits_{i=1}^kc_i\right)^{\gamma}\geq$
$\geq\left(\sum\limits_{i=1}^k\left(a_i^{\alpha}b_i^{\beta}c_i^{\gamma}\right)^{\frac{1}{\alpha+\beta+\gamma}}\right)^{\alpha+\beta+\gamma}$.
Всё это аналогично продолжается на любое число последовательностей положительных чисел.

Имеем:

$\sum\limits_{cyc}{\sqrt[3]{\frac{a}{b+1}}}=
\sum\limits_{cyc}(a+1)^\frac13\left(\frac{a}{a+1}\right)^\frac13\left(\frac{1}{b+1} \right)^\frac13 \leq \left(\sum\limits_{cyc}(a+1)\right)^\frac13\left(\sum\limits_{cyc}\frac{a}{a+1}\right)^\frac13\left(\sum\limits_{cyc}\frac{1}{b+1} \right)^\frac13=$

$=\sqrt[3]{9\sum\limits_{cyc}\frac{a}{a+1}\sum\limits_{cyc}\frac{1}{a+1}}\leq \sqrt[3]{9\left(\frac{\sum\limits_{cyc}\frac{a}{a+1}+\sum\limits_{cyc}\frac{1}{a+1}}2 \right)^2}=\sqrt[3]{36}$

 Профиль  
                  
Показать сообщения за:  Поле сортировки  
Начать новую тему Ответить на тему  [ Сообщений: 13 ] 

Модераторы: Модераторы Математики, Супермодераторы



Кто сейчас на конференции

Сейчас этот форум просматривают: YandexBot [bot]


Вы не можете начинать темы
Вы не можете отвечать на сообщения
Вы не можете редактировать свои сообщения
Вы не можете удалять свои сообщения
Вы не можете добавлять вложения

Найти:
Powered by phpBB © 2000, 2002, 2005, 2007 phpBB Group